Minimum $M=frac{4}{x^{2}}+frac{4}{y^{2}}+frac{1}{(x-y)^{2}}$ for $3x^2+8y^3=20$












0












$begingroup$



Let $x,y$ be positive real numbers such that $xneq y$ and
$3x^{2}+8y^{3}=20$. Find $x,y$ so that $M$ get the minimum value:
$M=frac{4}{x^{2}}+frac{4}{y^{2}}+frac{1}{(x-y)^{2}}$.




I can not find when this function gets the minimum value. Or can this difficult problem be solved by differentiating? How do we turn the root to another specie which has $ax+by$ or $frac{ax}{y}$ while $x,y$ are some constants.



Help me to solve this problem.










share|cite|improve this question











$endgroup$








  • 1




    $begingroup$
    What does > mean in the second line?
    $endgroup$
    – William Elliot
    Dec 13 '18 at 11:55
















0












$begingroup$



Let $x,y$ be positive real numbers such that $xneq y$ and
$3x^{2}+8y^{3}=20$. Find $x,y$ so that $M$ get the minimum value:
$M=frac{4}{x^{2}}+frac{4}{y^{2}}+frac{1}{(x-y)^{2}}$.




I can not find when this function gets the minimum value. Or can this difficult problem be solved by differentiating? How do we turn the root to another specie which has $ax+by$ or $frac{ax}{y}$ while $x,y$ are some constants.



Help me to solve this problem.










share|cite|improve this question











$endgroup$








  • 1




    $begingroup$
    What does > mean in the second line?
    $endgroup$
    – William Elliot
    Dec 13 '18 at 11:55














0












0








0


1



$begingroup$



Let $x,y$ be positive real numbers such that $xneq y$ and
$3x^{2}+8y^{3}=20$. Find $x,y$ so that $M$ get the minimum value:
$M=frac{4}{x^{2}}+frac{4}{y^{2}}+frac{1}{(x-y)^{2}}$.




I can not find when this function gets the minimum value. Or can this difficult problem be solved by differentiating? How do we turn the root to another specie which has $ax+by$ or $frac{ax}{y}$ while $x,y$ are some constants.



Help me to solve this problem.










share|cite|improve this question











$endgroup$





Let $x,y$ be positive real numbers such that $xneq y$ and
$3x^{2}+8y^{3}=20$. Find $x,y$ so that $M$ get the minimum value:
$M=frac{4}{x^{2}}+frac{4}{y^{2}}+frac{1}{(x-y)^{2}}$.




I can not find when this function gets the minimum value. Or can this difficult problem be solved by differentiating? How do we turn the root to another specie which has $ax+by$ or $frac{ax}{y}$ while $x,y$ are some constants.



Help me to solve this problem.







calculus inequality optimization nonlinear-optimization maxima-minima






share|cite|improve this question















share|cite|improve this question













share|cite|improve this question




share|cite|improve this question








edited Dec 13 '18 at 14:27









Martin Sleziak

44.8k10119272




44.8k10119272










asked Dec 13 '18 at 11:26









Trong TuanTrong Tuan

1318




1318








  • 1




    $begingroup$
    What does > mean in the second line?
    $endgroup$
    – William Elliot
    Dec 13 '18 at 11:55














  • 1




    $begingroup$
    What does > mean in the second line?
    $endgroup$
    – William Elliot
    Dec 13 '18 at 11:55








1




1




$begingroup$
What does > mean in the second line?
$endgroup$
– William Elliot
Dec 13 '18 at 11:55




$begingroup$
What does > mean in the second line?
$endgroup$
– William Elliot
Dec 13 '18 at 11:55










2 Answers
2






active

oldest

votes


















3












$begingroup$

Let $x=2$ and $y=1$.



Thus, we'll get a value $6$.



We'll prove that it's a minimal value.



Indeed, let $x=2a$ and $y=b$.



Thus, $3a^2+2b^3=5$ and we need to prove that
$$frac{1}{a^2}+frac{4}{b^2}+frac{1}{(2a-b)^2}geq6.$$
But by AM-GM
$$5=3a^2+2b^3=3a^2-1+3b^2+2b^3+1-3b^2geq$$
$$geq3a^2+3b^2-1+3sqrt[3]{(b^3)^2cdot1}-3b^2=3(a^2+b^2)-1,$$
which gives $2geq a^2+b^2.$



Id est, $$frac{1}{a^2}+frac{4}{b^2}+frac{1}{(2a-b)^2}geqfrac{1}{2}(a^2+b^2)left(frac{1}{a^2}+frac{4}{b^2}+frac{1}{(2a-b)^2}right).$$
Now, let $a=tb$.



Thus, we need to prove that
$$(t^2+1)left(frac{1}{t^2}+4+frac{1}{(2t-1)^2}right)geq12$$ or
$$(t-1)^2(16t^4+16t^3-7t^2-2t+1)geq0,$$ which is obviously true for $tgeq1.$



But for $0<tleq1$ we obtain:
$$16t^4+16t^3-7t^2-2t+1=$$
$$=9t^3(1-t)+left(5t^2+frac{7}{10}t-frac{5}{6}right)^2+frac{1}{900}(759t^2-750t+275)geq0.$$
Done!



Also, we can use derivative here.



Since $$(t^2+1)left(frac{1}{t^2}+4+frac{1}{(2t-1)^2}right)geq12$$ it's $f(t)geq0,$ where
$$f(t)=64t^6-64t^5-92t^4+112t^3-8t^2-16t+4,$$ and
$$f'(t)=(t-1)(t+1)(3t-1)(8t^2-4t-1),$$
we obtain $t_{min}=frac{1}{3}$ or $t_{min}=1$ and since $fleft(frac{1}{3}right)>f(1)=0$, we are done.






share|cite|improve this answer











$endgroup$













  • $begingroup$
    Can w use derivative to solve this. Thank you
    $endgroup$
    – Trong Tuan
    Dec 13 '18 at 13:28










  • $begingroup$
    @Trong Tuan I added something. See now.
    $endgroup$
    – Michael Rozenberg
    Dec 13 '18 at 13:50










  • $begingroup$
    In $$x=2,; y=1$$ is local minimum. Global minimum is in $x=-1.668453756732446,;y=1.133435720989714$
    $endgroup$
    – Aleksas Domarkas
    Dec 19 '18 at 11:38





















0












$begingroup$

Local minimum:
$$f(2,1)=6$$
Global minimum:
$$f(−1.668453756732446,1.133435720989714)=4.677920159716602$$
enter image description here






share|cite|improve this answer









$endgroup$













    Your Answer





    StackExchange.ifUsing("editor", function () {
    return StackExchange.using("mathjaxEditing", function () {
    StackExchange.MarkdownEditor.creationCallbacks.add(function (editor, postfix) {
    StackExchange.mathjaxEditing.prepareWmdForMathJax(editor, postfix, [["$", "$"], ["\\(","\\)"]]);
    });
    });
    }, "mathjax-editing");

    StackExchange.ready(function() {
    var channelOptions = {
    tags: "".split(" "),
    id: "69"
    };
    initTagRenderer("".split(" "), "".split(" "), channelOptions);

    StackExchange.using("externalEditor", function() {
    // Have to fire editor after snippets, if snippets enabled
    if (StackExchange.settings.snippets.snippetsEnabled) {
    StackExchange.using("snippets", function() {
    createEditor();
    });
    }
    else {
    createEditor();
    }
    });

    function createEditor() {
    StackExchange.prepareEditor({
    heartbeatType: 'answer',
    autoActivateHeartbeat: false,
    convertImagesToLinks: true,
    noModals: true,
    showLowRepImageUploadWarning: true,
    reputationToPostImages: 10,
    bindNavPrevention: true,
    postfix: "",
    imageUploader: {
    brandingHtml: "Powered by u003ca class="icon-imgur-white" href="https://imgur.com/"u003eu003c/au003e",
    contentPolicyHtml: "User contributions licensed under u003ca href="https://creativecommons.org/licenses/by-sa/3.0/"u003ecc by-sa 3.0 with attribution requiredu003c/au003e u003ca href="https://stackoverflow.com/legal/content-policy"u003e(content policy)u003c/au003e",
    allowUrls: true
    },
    noCode: true, onDemand: true,
    discardSelector: ".discard-answer"
    ,immediatelyShowMarkdownHelp:true
    });


    }
    });














    draft saved

    draft discarded


















    StackExchange.ready(
    function () {
    StackExchange.openid.initPostLogin('.new-post-login', 'https%3a%2f%2fmath.stackexchange.com%2fquestions%2f3037891%2fminimum-m-frac4x2-frac4y2-frac1x-y2-for-3x28y3-2%23new-answer', 'question_page');
    }
    );

    Post as a guest















    Required, but never shown

























    2 Answers
    2






    active

    oldest

    votes








    2 Answers
    2






    active

    oldest

    votes









    active

    oldest

    votes






    active

    oldest

    votes









    3












    $begingroup$

    Let $x=2$ and $y=1$.



    Thus, we'll get a value $6$.



    We'll prove that it's a minimal value.



    Indeed, let $x=2a$ and $y=b$.



    Thus, $3a^2+2b^3=5$ and we need to prove that
    $$frac{1}{a^2}+frac{4}{b^2}+frac{1}{(2a-b)^2}geq6.$$
    But by AM-GM
    $$5=3a^2+2b^3=3a^2-1+3b^2+2b^3+1-3b^2geq$$
    $$geq3a^2+3b^2-1+3sqrt[3]{(b^3)^2cdot1}-3b^2=3(a^2+b^2)-1,$$
    which gives $2geq a^2+b^2.$



    Id est, $$frac{1}{a^2}+frac{4}{b^2}+frac{1}{(2a-b)^2}geqfrac{1}{2}(a^2+b^2)left(frac{1}{a^2}+frac{4}{b^2}+frac{1}{(2a-b)^2}right).$$
    Now, let $a=tb$.



    Thus, we need to prove that
    $$(t^2+1)left(frac{1}{t^2}+4+frac{1}{(2t-1)^2}right)geq12$$ or
    $$(t-1)^2(16t^4+16t^3-7t^2-2t+1)geq0,$$ which is obviously true for $tgeq1.$



    But for $0<tleq1$ we obtain:
    $$16t^4+16t^3-7t^2-2t+1=$$
    $$=9t^3(1-t)+left(5t^2+frac{7}{10}t-frac{5}{6}right)^2+frac{1}{900}(759t^2-750t+275)geq0.$$
    Done!



    Also, we can use derivative here.



    Since $$(t^2+1)left(frac{1}{t^2}+4+frac{1}{(2t-1)^2}right)geq12$$ it's $f(t)geq0,$ where
    $$f(t)=64t^6-64t^5-92t^4+112t^3-8t^2-16t+4,$$ and
    $$f'(t)=(t-1)(t+1)(3t-1)(8t^2-4t-1),$$
    we obtain $t_{min}=frac{1}{3}$ or $t_{min}=1$ and since $fleft(frac{1}{3}right)>f(1)=0$, we are done.






    share|cite|improve this answer











    $endgroup$













    • $begingroup$
      Can w use derivative to solve this. Thank you
      $endgroup$
      – Trong Tuan
      Dec 13 '18 at 13:28










    • $begingroup$
      @Trong Tuan I added something. See now.
      $endgroup$
      – Michael Rozenberg
      Dec 13 '18 at 13:50










    • $begingroup$
      In $$x=2,; y=1$$ is local minimum. Global minimum is in $x=-1.668453756732446,;y=1.133435720989714$
      $endgroup$
      – Aleksas Domarkas
      Dec 19 '18 at 11:38


















    3












    $begingroup$

    Let $x=2$ and $y=1$.



    Thus, we'll get a value $6$.



    We'll prove that it's a minimal value.



    Indeed, let $x=2a$ and $y=b$.



    Thus, $3a^2+2b^3=5$ and we need to prove that
    $$frac{1}{a^2}+frac{4}{b^2}+frac{1}{(2a-b)^2}geq6.$$
    But by AM-GM
    $$5=3a^2+2b^3=3a^2-1+3b^2+2b^3+1-3b^2geq$$
    $$geq3a^2+3b^2-1+3sqrt[3]{(b^3)^2cdot1}-3b^2=3(a^2+b^2)-1,$$
    which gives $2geq a^2+b^2.$



    Id est, $$frac{1}{a^2}+frac{4}{b^2}+frac{1}{(2a-b)^2}geqfrac{1}{2}(a^2+b^2)left(frac{1}{a^2}+frac{4}{b^2}+frac{1}{(2a-b)^2}right).$$
    Now, let $a=tb$.



    Thus, we need to prove that
    $$(t^2+1)left(frac{1}{t^2}+4+frac{1}{(2t-1)^2}right)geq12$$ or
    $$(t-1)^2(16t^4+16t^3-7t^2-2t+1)geq0,$$ which is obviously true for $tgeq1.$



    But for $0<tleq1$ we obtain:
    $$16t^4+16t^3-7t^2-2t+1=$$
    $$=9t^3(1-t)+left(5t^2+frac{7}{10}t-frac{5}{6}right)^2+frac{1}{900}(759t^2-750t+275)geq0.$$
    Done!



    Also, we can use derivative here.



    Since $$(t^2+1)left(frac{1}{t^2}+4+frac{1}{(2t-1)^2}right)geq12$$ it's $f(t)geq0,$ where
    $$f(t)=64t^6-64t^5-92t^4+112t^3-8t^2-16t+4,$$ and
    $$f'(t)=(t-1)(t+1)(3t-1)(8t^2-4t-1),$$
    we obtain $t_{min}=frac{1}{3}$ or $t_{min}=1$ and since $fleft(frac{1}{3}right)>f(1)=0$, we are done.






    share|cite|improve this answer











    $endgroup$













    • $begingroup$
      Can w use derivative to solve this. Thank you
      $endgroup$
      – Trong Tuan
      Dec 13 '18 at 13:28










    • $begingroup$
      @Trong Tuan I added something. See now.
      $endgroup$
      – Michael Rozenberg
      Dec 13 '18 at 13:50










    • $begingroup$
      In $$x=2,; y=1$$ is local minimum. Global minimum is in $x=-1.668453756732446,;y=1.133435720989714$
      $endgroup$
      – Aleksas Domarkas
      Dec 19 '18 at 11:38
















    3












    3








    3





    $begingroup$

    Let $x=2$ and $y=1$.



    Thus, we'll get a value $6$.



    We'll prove that it's a minimal value.



    Indeed, let $x=2a$ and $y=b$.



    Thus, $3a^2+2b^3=5$ and we need to prove that
    $$frac{1}{a^2}+frac{4}{b^2}+frac{1}{(2a-b)^2}geq6.$$
    But by AM-GM
    $$5=3a^2+2b^3=3a^2-1+3b^2+2b^3+1-3b^2geq$$
    $$geq3a^2+3b^2-1+3sqrt[3]{(b^3)^2cdot1}-3b^2=3(a^2+b^2)-1,$$
    which gives $2geq a^2+b^2.$



    Id est, $$frac{1}{a^2}+frac{4}{b^2}+frac{1}{(2a-b)^2}geqfrac{1}{2}(a^2+b^2)left(frac{1}{a^2}+frac{4}{b^2}+frac{1}{(2a-b)^2}right).$$
    Now, let $a=tb$.



    Thus, we need to prove that
    $$(t^2+1)left(frac{1}{t^2}+4+frac{1}{(2t-1)^2}right)geq12$$ or
    $$(t-1)^2(16t^4+16t^3-7t^2-2t+1)geq0,$$ which is obviously true for $tgeq1.$



    But for $0<tleq1$ we obtain:
    $$16t^4+16t^3-7t^2-2t+1=$$
    $$=9t^3(1-t)+left(5t^2+frac{7}{10}t-frac{5}{6}right)^2+frac{1}{900}(759t^2-750t+275)geq0.$$
    Done!



    Also, we can use derivative here.



    Since $$(t^2+1)left(frac{1}{t^2}+4+frac{1}{(2t-1)^2}right)geq12$$ it's $f(t)geq0,$ where
    $$f(t)=64t^6-64t^5-92t^4+112t^3-8t^2-16t+4,$$ and
    $$f'(t)=(t-1)(t+1)(3t-1)(8t^2-4t-1),$$
    we obtain $t_{min}=frac{1}{3}$ or $t_{min}=1$ and since $fleft(frac{1}{3}right)>f(1)=0$, we are done.






    share|cite|improve this answer











    $endgroup$



    Let $x=2$ and $y=1$.



    Thus, we'll get a value $6$.



    We'll prove that it's a minimal value.



    Indeed, let $x=2a$ and $y=b$.



    Thus, $3a^2+2b^3=5$ and we need to prove that
    $$frac{1}{a^2}+frac{4}{b^2}+frac{1}{(2a-b)^2}geq6.$$
    But by AM-GM
    $$5=3a^2+2b^3=3a^2-1+3b^2+2b^3+1-3b^2geq$$
    $$geq3a^2+3b^2-1+3sqrt[3]{(b^3)^2cdot1}-3b^2=3(a^2+b^2)-1,$$
    which gives $2geq a^2+b^2.$



    Id est, $$frac{1}{a^2}+frac{4}{b^2}+frac{1}{(2a-b)^2}geqfrac{1}{2}(a^2+b^2)left(frac{1}{a^2}+frac{4}{b^2}+frac{1}{(2a-b)^2}right).$$
    Now, let $a=tb$.



    Thus, we need to prove that
    $$(t^2+1)left(frac{1}{t^2}+4+frac{1}{(2t-1)^2}right)geq12$$ or
    $$(t-1)^2(16t^4+16t^3-7t^2-2t+1)geq0,$$ which is obviously true for $tgeq1.$



    But for $0<tleq1$ we obtain:
    $$16t^4+16t^3-7t^2-2t+1=$$
    $$=9t^3(1-t)+left(5t^2+frac{7}{10}t-frac{5}{6}right)^2+frac{1}{900}(759t^2-750t+275)geq0.$$
    Done!



    Also, we can use derivative here.



    Since $$(t^2+1)left(frac{1}{t^2}+4+frac{1}{(2t-1)^2}right)geq12$$ it's $f(t)geq0,$ where
    $$f(t)=64t^6-64t^5-92t^4+112t^3-8t^2-16t+4,$$ and
    $$f'(t)=(t-1)(t+1)(3t-1)(8t^2-4t-1),$$
    we obtain $t_{min}=frac{1}{3}$ or $t_{min}=1$ and since $fleft(frac{1}{3}right)>f(1)=0$, we are done.







    share|cite|improve this answer














    share|cite|improve this answer



    share|cite|improve this answer








    edited Dec 13 '18 at 13:49

























    answered Dec 13 '18 at 12:04









    Michael RozenbergMichael Rozenberg

    106k1893198




    106k1893198












    • $begingroup$
      Can w use derivative to solve this. Thank you
      $endgroup$
      – Trong Tuan
      Dec 13 '18 at 13:28










    • $begingroup$
      @Trong Tuan I added something. See now.
      $endgroup$
      – Michael Rozenberg
      Dec 13 '18 at 13:50










    • $begingroup$
      In $$x=2,; y=1$$ is local minimum. Global minimum is in $x=-1.668453756732446,;y=1.133435720989714$
      $endgroup$
      – Aleksas Domarkas
      Dec 19 '18 at 11:38




















    • $begingroup$
      Can w use derivative to solve this. Thank you
      $endgroup$
      – Trong Tuan
      Dec 13 '18 at 13:28










    • $begingroup$
      @Trong Tuan I added something. See now.
      $endgroup$
      – Michael Rozenberg
      Dec 13 '18 at 13:50










    • $begingroup$
      In $$x=2,; y=1$$ is local minimum. Global minimum is in $x=-1.668453756732446,;y=1.133435720989714$
      $endgroup$
      – Aleksas Domarkas
      Dec 19 '18 at 11:38


















    $begingroup$
    Can w use derivative to solve this. Thank you
    $endgroup$
    – Trong Tuan
    Dec 13 '18 at 13:28




    $begingroup$
    Can w use derivative to solve this. Thank you
    $endgroup$
    – Trong Tuan
    Dec 13 '18 at 13:28












    $begingroup$
    @Trong Tuan I added something. See now.
    $endgroup$
    – Michael Rozenberg
    Dec 13 '18 at 13:50




    $begingroup$
    @Trong Tuan I added something. See now.
    $endgroup$
    – Michael Rozenberg
    Dec 13 '18 at 13:50












    $begingroup$
    In $$x=2,; y=1$$ is local minimum. Global minimum is in $x=-1.668453756732446,;y=1.133435720989714$
    $endgroup$
    – Aleksas Domarkas
    Dec 19 '18 at 11:38






    $begingroup$
    In $$x=2,; y=1$$ is local minimum. Global minimum is in $x=-1.668453756732446,;y=1.133435720989714$
    $endgroup$
    – Aleksas Domarkas
    Dec 19 '18 at 11:38













    0












    $begingroup$

    Local minimum:
    $$f(2,1)=6$$
    Global minimum:
    $$f(−1.668453756732446,1.133435720989714)=4.677920159716602$$
    enter image description here






    share|cite|improve this answer









    $endgroup$


















      0












      $begingroup$

      Local minimum:
      $$f(2,1)=6$$
      Global minimum:
      $$f(−1.668453756732446,1.133435720989714)=4.677920159716602$$
      enter image description here






      share|cite|improve this answer









      $endgroup$
















        0












        0








        0





        $begingroup$

        Local minimum:
        $$f(2,1)=6$$
        Global minimum:
        $$f(−1.668453756732446,1.133435720989714)=4.677920159716602$$
        enter image description here






        share|cite|improve this answer









        $endgroup$



        Local minimum:
        $$f(2,1)=6$$
        Global minimum:
        $$f(−1.668453756732446,1.133435720989714)=4.677920159716602$$
        enter image description here







        share|cite|improve this answer












        share|cite|improve this answer



        share|cite|improve this answer










        answered Dec 19 '18 at 20:43









        Aleksas DomarkasAleksas Domarkas

        1,37216




        1,37216






























            draft saved

            draft discarded




















































            Thanks for contributing an answer to Mathematics Stack Exchange!


            • Please be sure to answer the question. Provide details and share your research!

            But avoid



            • Asking for help, clarification, or responding to other answers.

            • Making statements based on opinion; back them up with references or personal experience.


            Use MathJax to format equations. MathJax reference.


            To learn more, see our tips on writing great answers.




            draft saved


            draft discarded














            StackExchange.ready(
            function () {
            StackExchange.openid.initPostLogin('.new-post-login', 'https%3a%2f%2fmath.stackexchange.com%2fquestions%2f3037891%2fminimum-m-frac4x2-frac4y2-frac1x-y2-for-3x28y3-2%23new-answer', 'question_page');
            }
            );

            Post as a guest















            Required, but never shown





















































            Required, but never shown














            Required, but never shown












            Required, but never shown







            Required, but never shown

































            Required, but never shown














            Required, but never shown












            Required, but never shown







            Required, but never shown







            Popular posts from this blog

            Plaza Victoria

            In PowerPoint, is there a keyboard shortcut for bulleted / numbered list?

            How to put 3 figures in Latex with 2 figures side by side and 1 below these side by side images but in...